Jan is saving money to buy a bike. The bike costs $255. She has $120 saved and each week she adds $15 to her savings. Write and solve an equation to find how long will it take her to save enough money to buy the bike.

Answers

Answer 1
Answer is 9 weeks

Step by step

We can use the y intercept formula
y = mx + b

The cost of the bike is “y” $255
The starting point is “b” $120 saved
The rate is “m” $15 per week
Our unknown amount of weeks is x

$255 = $15x + $120

Solve for x

Subtract 120 from both sides to isolate x

255 - 120 = 15x + 120 - 120

135 = 15x

Divide both sides by 15 to solve for x

135 /15 = 15/15 x

9 = x

9 = amount of weeks

We can check our work by substituting 9 in place of x in our equation

255 = (15)(9) + 120

255 = 135 + 120

255 = 255

Related Questions

What is the probability of spinning B on the first spinner and an even number on the second spinner?

Answers

Answer:

[tex]\frac{2}{15}[/tex]

Step-by-step explanation:

On the first spinner, there are 3 possible outcomes. Of those 3 outcomes, only one is what we're looking for (Landing on B). This means that the chance to land on B on the first spinner is [tex]\frac{1}{3}[/tex].

For the second spinner, There a 5 possible outcomes, and 2 even numbers on the spinner, meaning the chance to land on an even number is [tex]\frac{2}{5}[/tex].

Since these are independent events, we can simply multiply these numbers together and get [tex]\frac{1}{3}*\frac{2}{5}=\frac{2}{15}[/tex].

Translate the sentence into an equation.
Nine more than the product of a number and 6 is equal to 4.
Use the variable x for the unknown number.

Answers

Answer:

(6x) + 9 = 4

Step-by-step explanation:

Times means 'The answer when you multiply' So we are working in multiplication. ( x )

6x is the multiplication.

It says 'more than' which means adding to our answer. So we add 9. So far our equation is: (6x) + 9 (Brackets are needed as we are multiplying 6 by x and not multiplying 6 by x+9)

Lastly, it says the answer must be 4 so we add that at the end. So the answer is:

(6x) + 9 = 4

please help me with this

Answers

the correct option is the first one (counting from the top).

The true statement is:

"The leading coefficients of f(x) and g(x) are opposites"

Which statement is true?

We know that f(x) and g(x) are quadratic functions, and the sum of these two functions is shown in the graph.

We can see that the sum gives a linear graph, so the terms with the exponents of 2 are cancelled by the sum.

This only can happen if f(x) and g(x) have opposite (same absolute value but different sign) leading coefficients, so the quadratic terms are cancelled when we add the functions.

For example in f(x) = 2*x^2 + 3x + 4

                         g(x) = -2x + x + 13

If we add these two functions, we would end with a line.

With that in mind we can conclude that the correct option is the first one (counting from the top).

If you want to laern more about quadratic equations:

https://brainly.com/question/1214333

#SPJ1

Select all the triangles that have an area of 10 square units:

Answers

The triangles that have an area of 10 square units are triangles A and B

How to determine the triangles that have an area of 10 square units?

The area of a triangle is calculated using

Area = 0.5 * Base * Height

Triangle 1

Base = 5

Height = 4

So, we have

Area = 0.5 * 5 * 4

Area = 10

Triangle 2

Base = 4

Height = 5

So, we have

Area = 0.5 * 5 * 4

Area = 10

Triangle 3

Base = 5

Height = 5

So, we have

Area = 0.5 * 5 * 5

Area = 12.5

Hence, the triangles that have an area of 10 square units are triangles A and B

Read more about areas at:

https://brainly.com/question/19819849

#SPJ1

In Euclidean geometry, the interior angles of any triangle add up to 180 degrees. Does this rule apply to triangles in spherical geometry? If not, is there another value that they would always sum to? Explain your answer.

Answers

The triangles are subject to this law. If another number were present, their sum would always be greater than 180 degrees.

What is geometry?

It is defined as the branch of mathematics that is concerned with the size, shape, and orientation of two-dimensional figures.

It is given that:

In Euclidean geometry, the interior angles of any triangle add up to 180 degrees.

As we know,

Spherical trigonometry, which departs from conventional trigonometry in many ways, is created when angles between great circles are defined.

For instance, the total of the sides and angles of a hemispheric triangle is more than 180 degrees.

In spherical geometry, triangles are subject to this law. If another number were present, their sum would always be greater than 180 degrees.

Thus, the triangles are subject to this law. If another number were present, their sum would always be greater than 180 degrees.

Learn more about geometry here:

brainly.com/question/16836548

#SPJ1

8/36 = /9 what number is missing to equal 9

Answers

Answer:

72

Step-by-step explanation:

I am pretty certain that the answer is 72.

Steps:

[tex]\frac{8}{36}[/tex] [tex]\frac{x}{9}[/tex]

Cross-Multiply and Divide.

8*9 = 72

72 / 36 = 2

2/9 = 0.222222

8/36 = 0.222222

2/9 = 0.2222222

2x+1=2x-1 what is x equaled too

Answers

Answer:

x ≤ 0 = no solution

Step-by-step explanation:

[tex]\sf (-2x)2x+1-2x=2x-1-2x\\1=-1\\= 0\\No~solution.[/tex]

Let f(X)=3√x
if g(x) is the graph of the f(x) shifted down 1 units and left 5 units write a formula for g(x)
g(x)=

Answers

g(x) = 3√(x-5) -1

The process of altering a graph to produce a different version of the preceding graph is known as graph transformation. The graphs can be moved about the x-y plane or translated. They may also be stretched, or they may undergo a mix of these changes.

Horizontal stretching: It means the graph is elongated or shrink in x direction.

Vertical stretching : It means the graph is elongated or shrink in y direction

Vertical translation : It means moving the base of the graph in y direction

Horizontal translation : It means moving the base of the graph in x direction

According to rules of transformation f(x)+c shift c units up and f(x)-c shift c units down.

Therefore, in order to  move the graph down 1 units, we need to subtract given function by 1 , we get

g(x) = 3√x -1

According to rules of transformation f(x+c) shift c units left and f(x-c ) shift c units right.

Therefore, in order to  move the graph left by 5 units, we need to add given function by 5 , we get

g(x) = 3√(x-5) -1

To learn more about graphical transformation,  refer to https://brainly.com/question/4025726

#SPJ9

5. This diagram is a straightedge and
compass construction. Which triangle is
equilateral? Explain how you know.

Answers

The ΔVZW is Equilateral Triangle .

Equilateral Triangle is defined as the triangle in which all the three sides of the triangle are equal.

For Example ; if a triangle ABC has lengths AB=2cm , BC=2cm and CA=2cm    then the triangle ABC can be called as an equilateral triangle as all sides have same length i.e. 2cm .

In the question ,

From the diagram, we have the radius of all the circles are of equal length ,

Since the radius of all the circles are equal in length , their diameter will also be equal in length .

We can see from the diagram that VZ , ZW and WV are the diameters of the circles , So they will be equal in length ,

which means VZ = ZW = WV.

In triangle VZW the sides VZ , ZW and WV  are of equal length , hence the triangle will be an Equilateral Triangle.

Therefore , the Triangle VZW is Equilateral Triangle .

Learn more about Equilateral Triangle here

https://brainly.com/question/12608133

#SPJ1

negative 19 plus the quantity negative 4 and five tenths plus 6 and 87 hundredths end quantity divided by 3 all times 5 squared minus 7 and 3 tenths
19.6
−18.4
31.45
−6.55

Answers

Answer: 19.6

Step-by-step explanation:

What is the solution to this equation?
x+ 14 = -6
OA. x= -8
OB. x=-20
OC. X = 20
OD. x=8

Answers

Answer: -20

Explanation:
X + 14 = -6
X -14 = -14
X = -20
X=-6-14
X=-20
OB is the correct answer

SO
129193ni-algmiz srit of
Kings Park Bank is advertising a special 1.66% APR for CDs. Kevin takes out a one-year CD
$24,000. The interest is compounded daily. Find the APY for Kevin's account.
wh
Sissy ono to brie ed is thus torni aver verdha lliw roum woh
5.00%

Answers

APY for Kevin's account = 1.67%

The interest on a loan or deposit that is computed based on both the original principle and the accumulated interest from prior periods is known as compound interest (also known as compounding interest). Compared to simple interest, which is computed just on the principal amount, it will cause a sum to increase more quickly.

The frequency of compounding determines the pace at which compound interest accumulates. The compound interest increases with the number of compounding periods.

APY = [tex](1 + \frac{r}{n})^{n} - 1[/tex]  =   [tex](1 + \frac{1.667}{365})^{365} - 1[/tex]  =  1.0167 - 1 = 0.0167

APY = 1.67%

Therefore, APY for Kevin's account = 1.67%

To know more about Compound Interest, refer to this link :

https://brainly.com/question/14295570

#SPJ1

What is 6.4% written as a decimal number? 64.0 6.4 0.64 0.064

Answers

Answer:

0.064

Step-by-step explanation:

To convert just muiltiply the percent by 100.

pls answer if wrong i ban you

Answers

The coordinates of L are: (8,-4)

Here, triangle is rotated 270° counterclockwise. So the rule that we have to apply here is

(x, y) → (y, -x)

Based on the rule given in step 1, we have to find the vertices of the rotated figure.

(x, y) → (y, -x)

L(4,8) → L'(8.-4)

M(4,3) → M'(3,-4)

N(8,8) → N'(8,-8)

Vertices of the rotated figure are:

L'(8.-4)  M'(3,-4)  N'(8,-8)

 

Hence we get the desired result.

Learn more about Coordinate geometry here:

brainly.com/question/12959377

#SPJ9

Help don’t understand question

Answers

C. Robert gets to keep a $100

A quadratic function is decreasing on (-∞, 3) and increasing on (3, ∞).
Will the vertex be the highest or lowest point on the graph of the parabola?
Explain.

Answers

Greetings from Brasil...

If X grows and Y grows, the function is increasing

If X increases and Y decreases, the function is decreasing

Besides that, according to the statement, we can conclude that the parabola has an upward concavity, that is, a > 0. In this case the vertex will be a minimum point

check the attachments

P.S. - vertex coordinate = (3; 0)

A farmer can plow a given field in 11 hours. If his son helps him, they can plow the vehicle together in 7 hours. How many hours would it take his son to plow the field alone? Express your answer as a fraction reduced to lowest terms, if needed

Answers

The total number of hours taken by farmer's son to plow the farm is 19 hours.

What is the concept of time and work?

Time refers to the duration of any task or activity that occurs or continues.

Work is a process or set of tasks designed to achieve a specific result.

We can define work happening as follows: - If a person A completes a task in X days, the amount of work finished in 1 day is 1/x.Likewise, if a person B finished work in Y days, his work completed in 1 day will become 1/Y.We can conclude from the preceding two points that A and B can complete (1/(X+Y) amount of work in one day. As a result, A and B could really complete the task in X Y/(X+Y) days.

For, the given question;

Let F be the number of hours taken by farmer to finish the work.

Let S be the number of hours taken by farmer' son to finish the work.

The amount of work finished in 1 hour by farmer is 1/F = 1/11.

The amount of work finished in 1 hour by farmer's son is 1/S.

The amount of work finished by both in 1 hours is;

1/F + 1/S = 1/7

1/S = 1/7 - 1/F

1/S = 1/7 - 1/11

Simplifying the given term;

1/S = 4/77

or, S = 77/4 = 19.25 hours.

S = 19 hours (fraction reduced to lowest terms)

Thus, the time taken by the farmer's son to complete the plow of field is 19 hours.

To know more about the time and work, here

https://brainly.com/question/19382734

#SPJ9

find the distance between the two points in simplest radical form. (2,6) and (7,-6)

Answers

Answer:

13

Step-by-step explanation:

Use the distance formula to determine the distance between two points.
√[(x₂ - x₁)² + (y₂ - y₁)²]

for small changes in temperature, the formula for the expansion of a metal rod under a change in temperature is: g-l=al(t-t), where g is the length of the object at temperature t, and l is the length at temperature t, and a is a constant which depends on the type of metal. a rod is 135 cm long at 115 degrees f and made of a metal with a=10^-3. write an equation giving the length of this rod at temperature t(because a=10^-3 is very small, give an exact answer: do not round expand multiplied numerical products to get rounded decimals).

Answers

Equation giving the length of this rod at temperature is g = 0.000085t + 84.991075 cm.

What is temperature ?

A temperature is a unit used to represent voguishness or coolness on any of a number of scales, including Fahrenheit and Celsius. According to temperature, heat energy will naturally move from a hotter( body with an advanced temperature) to a colder( body with a lower temperature)( one at a lower temperature).

Given that,

Temperature denoted by T

G - L = aL(t - T)

G = aLt +  L(1 - aT)

Slope = aL

Intercept = L(1 - aT)

g = 0.000085t + 84.991075 cm

Learn more about temperature here: https://brainly.com/question/24746268

#SPJ9

How many times smaller is 5×10-7 than 8.5×10-4

Answers

Answer:

Approximately 1.86 times smaller

Step-by-step explanation:

5×10-7 = (5×10=50) = 50-7 = 43

8.5×10-4 = (8.5×10=84) = 84- 4 = 80

80/43 = approximately 1.86

43×1.86 = approximately 80

80/1.86 = approximately 43

About 5 times per size

Kenna has a 80.3 foot length of pipe to cut into pieces. Each piece must be 9.4 feet long. How many pieces of this length can Kenna get from this one length of pipe?

Answers

Step-by-step explanation:

80.3÷9.4

80/3÷9/4

80/3×4/9

26.6×4/9

=17.733

This means that Kenna will get 18 pieces from one lenght of pipe

Please answer quick!

Answers

Answer:

-67

Step-by-step explanation:

[tex]x=-20 \implies x<-12 \\ \\ \therefore f(-20)=3(-20)-7=-67[/tex]

make a research about fibonacci sequence and its process kindly give at least five examples.

Answers

What is the Fibonacci sequence?

The Fibonacci sequence is a set of integers (the Fibonacci numbers) that starts with a zero, followed by a one, then by another one, and then by a series of steadily increasing numbers. The sequence follows the rule that each number is equal to the sum of the preceding two numbers.

The Fibonacci sequence begins with the following 14 integers:

0, 1, 1, 2, 3, 5, 8, 13, 21, 34, 55, 89, 144, 233 ...

Each number, starting with the third, adheres to the prescribed formula. For example, the seventh number, 8, is preceded by 3 and 5, which add up to 8.

The sequence can theoretically continue to infinity, using the same formula for each new number. Some resources show the Fibonacci sequence starting with a one instead of a zero, but this is fairly uncommon.

Example 1: Starting with 0 and 1, write the first 5 Fibonacci numbers.

Solution: The formula for the Fibonacci sequence is Fn= Fn-1+Fn-2

The first and second terms are 0 and 1, respectively.

F0 = 0 and F1 = 1.

F2 = F0 + F1 = 0+1 = 1 is the third term.

F3 = F2+F1 = 1 + 1 = 2 is the fourth term.

F4 = F3+F2 = 1+2 = 3 is the fifth term.

The Fibonacci sequence’s first five terms are 0,1,1,2,3.

Example 2: Find the Fibonacci series’ next term: 0, 1, 1, 2, 3, 5…

Solution: The preceding two Fibonacci terms are the next term in the series.

As a result, the word that must be used is – 3 + 5 = 8

Example 3: What is the significance of Fibonacci numbers?

Solution: In financial statement analysis, Fibonacci numbers are pretty helpful. In addition, the Fibonacci number series may be utilized to provide valuable proportions or ratios for business people.

Example 4: Find the 12th term of the Fibonacci sequence if the 10th and 11th terms are 34 and 55, respectively.

Solution: Using the Fibonacci Sequence recursive formula, we can say that the 12th term is the sum of the 10th term and the 11th term

12th term = 10th term + 11th term

= 34 + 55

= 89

The 12th term of the Fibonacci sequence is 89.

Example 5: The 14th term in the sequence is 377. Find the next term.

Solution: We know that the 15th term = the 14th term × the golden ratio.

F15 = 377 × 1.618034

≈ 609.99 = 610

The 15th term in the Fibonacci sequence is 610.

learn more of Fibonacci sequence here https://brainly.com/question/14771443

#SPJ9

Please help ASAP!!

Reporting wring answers. Thank you.

Write a problem that uses both multiplication and division
and whose answer is -9. Write the problem and then show the solution.

Answers

Answer:

(-27*3) ÷9

Step-by-step explanation:

-27*3=-81

-81/9=-9

guys!! please help me, I’ll give out brainliest just please help it’s my final question and it’s currently 4 in the morning .

(geometry)
what is the perimeter of the rectangle ABCD?

Answers

Answer:

Please disregard

Step-by-step explanation:

The box plot represents the distribution of speeds, in miles per hour, of 100 cars as they passed through a busy intersection.

What is the first quartile? Interpret this value in the situation.

About *insert fraction* of the cars going through the intersection were going *Q1* or slower.

Answers

Answer:

Step-by-step explanation:

Given,

N= 100

Q1=?

We know that,

Q1= N/4

   = 100/4

   = 25

Help please asap i need it

Answers

Hi
Check out the attached photo

There are 20 flowers in a bouquet, and 2 of them are roses. What percent of the flowers are roses?

Answers

Answer:

10%

Step-by-step explanation:

firstly express as a fraction then multiply the fraction by 100% , that is

[tex]\frac{2}{20}[/tex] × 100% = 2 × 5 = 10% of the flowers are roses

(z^1)^2

Its Khan Academy I need help ASAP.

Answers

z^2

Simplify step - by - step

( z^1 )^2

( a^m )^n = a^mn

z^1•2

z^2

PLEASE SOLVE THIS FOR ME. I ONLY KNOW THAT THE ANSWER WILL HAVE AN X IN IT

Answers

Answer: [tex]\boxed{\sqrt{12x-1}}[/tex]

Step-by-step explanation:

[tex]\sqrt{2(6x-3)+5}\\ \\\sqrt{(12x-6)+5} \\\\\boxed{\sqrt{12x-1}} \\\\This\ is\ as\ far\ as\ you\ can\ simplify\ this[/tex]

The answer to this is f = √12x-1 / gx

Hope this helps! :)
Other Questions
Which type of malware relies on lolbins? a. pup b. bot c. file-based virus d. fileless virus Large-denomination cds are ________, so that like a bond, they have a ________degree of liquidity and can be sold in secondary markets. Why did so many outlaws operate in the American West? The need for positive self-regard is ____; the drive toward self-enhancement is ____. Open the file to answer the question School fees extend has the government contributed to help or stop social issue to ease the burden for ordinary south africans ? Universal containers (uc) is concerned that data is being corrupted daily either through negligence or maliciousness. they want to implement a backup strategy to help recover any corrupted data or data mistakenly changed or even deleted. what should the data architect consider when designing a field -level audit and recovery plan? a. reduce data storage by purging old data. b. implement an appexchange package. c. review projected data storage needs. d. schedule a weekly export file. Which intervention would the nurse perform to teach the patient how to self-administer insulin? Select the correct answers.Which of these texts uses the most informal vocabulary?Students are encouraged to visit the college's campus before thestart of theircourses. This allows them to find their way SO that their first day runs smoothly.Students! Take a look around the college's grounds andsee what's what. Find outabout your course, the history of the college, and all the different student groups youcan join. It's a great way to start as you mean to go on!Would-be students are more than welcome to pop along to the college's open eveningto take a gander around and meet some new friends! Which variable differs systematically for the two distances that requires us to consider the rate of diffusion for these two layers separately? Creating a formula by _____ to the specific cells that are to be part of the formula is more accurate than typing the cell references. What is a first step to solve the equation 0.3n - 15 = 0.2n - 5? 2Active39Read the excerpt from Act I, scene i of Romeo andJuliet.Romeo: Alast that love, whose view is muffled still,Should, without eyes, see pathways to his will.160Where shall we dine? O mel What fray was here?Yet tell me not, for I have heard it all.Here's much to do with hate, but more with love:Why then, O brawling lovel O loving hatelO any thing! of nothing first create.O heavy lightness! serious vanity!Mis-shapen chaos of well-seeming formstFeather of lead, bright smoke, cold fire, sickMark this and return165Which lines from the excerpt support the inferencethat Romeo is emotionally conflicted? Select 3options.TIME REMAINING27:01Should, without eyes, see pathways to his willYet tell me not, for I have heard it allO heavy lightnesst serious vanity!4Mis-shapen chaos of well-seeming formstOFeather of lead, bright smoke, cold fire, sick health!Dost thou not laugh? 5000 nails in five boxes. the first and second boxes have 2700 nails all together. the second and the third boxes have 2000 nails all together. the third and fourth boxes have 1800 nails all together. the fourth and the fifth boxes have 1700 nails all together. how many nails are in each box Why is sugar solution classified the way it is? After a sale is made and consumers have purchased the product or service from a company, which additional imc tools are used to support them, keep them satisfied, and retain them as customers? Benedict has worked for the local utility company for over 40 years. With his maturity and business experience, his decisions are sound, based on solid, proven values. Benedict tends to be an independent thinker who focuses on empowering his employees. Kohlberg would probably think benedict is at the ________ level of personal moral development. Help me ASP SHOW UR WORK THANK YOU!! Suppose that before the stock market crash of 1929, the government taxed at an optimal rate. the stock market crash would have shifted the laffer curve. About 70% of people are right-handed. Of 352students in a school. Which is a reasonable number of right-handed students ?